LSAT and Law School Admissions Forum

Get expert LSAT preparation and law school admissions advice from PowerScore Test Preparation.

User avatar
 Dave Killoran
PowerScore Staff
  • PowerScore Staff
  • Posts: 5853
  • Joined: Mar 25, 2011
|
#27301
Complete Question Explanation
(The complete setup for this game can be found here: lsat/viewtopic.php?t=11508)

The correct answer choice is (C)

If J is scheduled for the third day, then only the first three possibilities shown in the setup discussion apply. In a Could Be True question, look for the uncertainty, which occurs in possibilities 1 and 2 with H and U. Only answer choice (C) includes H and U, and, as it is possible for H’s lesson to be scheduled for a later day than U’s lesson, answer choice (C) is correct.

Get the most out of your LSAT Prep Plus subscription.

Analyze and track your performance with our Testing and Analytics Package.